A function bounded and differentiable, but have an unbounded derivative?

Click For Summary
A function can be bounded and differentiable while having an unbounded derivative, particularly when the domain is chosen appropriately. An example involves a function whose derivative approaches infinity while remaining bounded, such as a monomial or a function resembling a periodic graph with decreasing peak distances. The discussion emphasizes the importance of providing hints rather than complete solutions in educational settings. The square root function on a restricted interval is also mentioned as a potential example. Overall, the exploration of such functions highlights interesting properties of calculus and function behavior.
danielkyulee
Messages
5
Reaction score
0
Can a function f: (a,b) in R be bounded and diffferentiablle, but have an unbounded derivative. I believe it can, but can not think of any examples where this is true. Anyone have any ideas?
 
Physics news on Phys.org
Sure, so long as you properly choose your domain.

So let's think about this, we want a function whose derivative approaches infinity, but such that the function itself is bounded. Probably the easiest way to do this is to find a function whose derivative goes to zero, then rotate about the axis y=x.

You don't need anything complicated. A monomial will do fine.
 
Try to think of a function with a graph that looks a bit like the graph of a periodic function, but is such that the distance between the peaks get shorter and shorter instead of staying the same.
 
Fredrik said:
Try to think of a function with a graph that looks a bit like the graph of a periodic function, but is such that the distance between the peaks get shorter and shorter instead of staying the same.

Very nice. Perhaps somewhat more complicated, but nice.
 
Sqrt(x), 0 < x < 1.
 
Ray, here in the homework forum it's important to answer with hints, not complete solutions.
 
Question: A clock's minute hand has length 4 and its hour hand has length 3. What is the distance between the tips at the moment when it is increasing most rapidly?(Putnam Exam Question) Answer: Making assumption that both the hands moves at constant angular velocities, the answer is ## \sqrt{7} .## But don't you think this assumption is somewhat doubtful and wrong?

Similar threads

  • · Replies 3 ·
Replies
3
Views
3K
Replies
3
Views
2K
  • · Replies 3 ·
Replies
3
Views
2K
  • · Replies 2 ·
Replies
2
Views
2K
  • · Replies 2 ·
Replies
2
Views
1K
  • · Replies 8 ·
Replies
8
Views
2K
  • · Replies 2 ·
Replies
2
Views
1K
  • · Replies 10 ·
Replies
10
Views
2K
  • · Replies 16 ·
Replies
16
Views
3K
Replies
26
Views
2K